Đến nội dung

trananh2771998 nội dung

Có 113 mục bởi trananh2771998 (Tìm giới hạn từ 25-04-2020)



Sắp theo                Sắp xếp  

#482818 $f(x+y)+f(x-y)=2f(x)+2f(y)$

Đã gửi bởi trananh2771998 on 12-02-2014 - 22:05 trong Phương trình hàm

Cho x=y ta được f(2x)=4f(x)

Cho x=2y ta được f(3x)=9f(x)

Cho x=y=0 ta được f(0)=0

Ta chứng minh quy nạp : f(nx)=$n^{2}$f(x) (1) ($n\epsilon N*$)

Với n=1 đúng

Giả sử đúng tới n .Ta có :

f((n+1)x)=f(nx+x)= -f((n-1)x) + 2f(nx) +2f(x) =$-(n-1)^{2}f(x)+ 2n^{2}f(x)+2f(x)=(n+1)^{2}f(x)$

Vậy (1) cũng đúng với n+1 ($n\epsilon N*$)

Từ đó ta có : f(1)=f($n\frac{1}{n}$)=$n^{2}f(\frac{1}{n})$ suy ra f($\frac{1}{n}$)=$\frac{f(1)}{n^{2}}$

tương tự f(m)=$n^{2}f(\frac{m}{n})$ suy ra $f(\frac{m}{n})=\frac{f(m)}{n^{2}}=\frac{m^{2}}{n^{2}}f(1)$

Tóm lại : f($\frac{m}{n}$)=($(\frac{m}{n})^{2}$)f(1)

Đặt f(1)=a suy ra f(x)=a$x^{2}$ với x>0 và x thuộc Q

với x<0 suy ra f(x)=f(-x)=a$(-x)^{2}$=a$(x)^{2}$

Vì f(0)=0 nên f(x)=a$x^{2}$

Thử lại thấy thoả mãn.Vậy f(x)=a$x^{2}$ với x thuộc Q với a là hằng số ($a\epsilon Q$) :icon6: :icon6: :icon6:




#474866 Tìm nghiệm nguyên dương của phương trình: $2^{x}+1=y^{2...

Đã gửi bởi trananh2771998 on 02-01-2014 - 22:25 trong Số học

Ta có :$2^x=(y-1)(y+1)= > y-1=2^m,y+1=2^n= > 2^n-2^m=2= > 2^m(2^{n-m}-1)=1.2= > m=0,n=1= > y=1,x=1$

Mình thấy x=y=1 có thỏa mãn đâu




#466380 China TST 2011

Đã gửi bởi trananh2771998 on 23-11-2013 - 23:08 trong Số học

http://www.artofprob...6a1f68#p2508288

Hình như ở trang cũng có một lời giải




#454239 Đề thi chọn học sinh giỏi toán lớp 11-12 chuyên KHTN 2013-2014 (Vòng 1)

Đã gửi bởi trananh2771998 on 30-09-2013 - 15:32 trong Thi HSG cấp Tỉnh, Thành phố. Olympic 30-4. Đề thi và kiểm tra đội tuyển các cấp.

Lời giải. Đặt $A=a^5$. Áp dụng bổ đề như bài này rồi ta suy ra $a-1 \equiv 1 \pmod{p}$ và $a^4+a^3+a^2+a+1 \equiv 1 \pmod{p}$ suy ra $p|30$. Do đó $p \in \{ 2;3;5 \}$.

P=31 cũng được mà Toàn.Cho $n=n^{2}=...=n^{p-1}=1$ Khi ấy A=32 :icon6:




#441194 Giúp mình chọn trường

Đã gửi bởi trananh2771998 on 08-08-2013 - 09:35 trong Các môn tự nhiên (Vật lý, Hóa học, Sinh học, Công nghệ)

Nếu bạn muốn nghiên cứu thì vote cho KHTN :icon6: :icon6: :icon6: :icon6:




#441121 Giúp mình chọn trường

Đã gửi bởi trananh2771998 on 07-08-2013 - 20:48 trong Các môn tự nhiên (Vật lý, Hóa học, Sinh học, Công nghệ)

1 Phiếu cho ĐH Sư Phạm :icon4: :icon4: :icon4: :icon4: :icon4: :icon4: :icon4:




#439585 Những bài hát bất hủ

Đã gửi bởi trananh2771998 on 31-07-2013 - 22:38 trong Quán nhạc

Hình như nhóm này rất nổi tiếng ở Việt Nam.Nhóm hình như tên HKT thì phải :ohmy:




#437000 $p_{1}p_{2}...p_{n}+1$

Đã gửi bởi trananh2771998 on 21-07-2013 - 22:23 trong Số học

Bạn lấy đâu ra các giả thiết này vậy :huh:




#435897 Giải PT: a)$x^2+\sqrt{2-x}=2x^2\sqrt{2-x}...

Đã gửi bởi trananh2771998 on 17-07-2013 - 22:22 trong Phương trình, hệ phương trình và bất phương trình

uhm tớ thấy cũng hơi có vấn đề nhưng thay vào đúng cậu ạ

X= 1 thì đúng rồi nhưng cái chỗ bôi đỏ nó cứ như thế nào ấy




#435894 Giải PT: a)$x^2+\sqrt{2-x}=2x^2\sqrt{2-x}...

Đã gửi bởi trananh2771998 on 17-07-2013 - 22:14 trong Phương trình, hệ phương trình và bất phương trình

Đặt $x^{2}=a;\sqrt{2-x}=b$

$\Rightarrow a+b=2ab$

Theo AM-GM ta có $ab\leq 1$

$\Rightarrow a+b> 2\sqrt{ab}\geq 2ab$

$\Rightarrow x=1$

Chỗ này mình thấy có vấn đề bạn ạ




#435727 Ban nick ?

Đã gửi bởi trananh2771998 on 16-07-2013 - 22:12 trong Góp ý cho diễn đàn

Tội nghiệp thật :off:




#435533 Tìm tất cả các số nguyên tố p

Đã gửi bởi trananh2771998 on 15-07-2013 - 22:45 trong Số học

Tìm tất cả các số nguyên tố p sao cho tồn tại các số nguyên dương n,x,y thoả mãn

$p^{n}= x^{3}+y^{3}$




#434704 $\large \sum \frac{a^{4}}{b+3c...

Đã gửi bởi trananh2771998 on 12-07-2013 - 11:15 trong Bất đẳng thức và cực trị

Mình vẫn thắc mắc chỗ màu xanh

Theo C-S thì $\left ( \sum a^{2} \right )\left ( 1+1+1 \right )\geq \left ( \sum a \right )^{2}\geq 3\left ( \sum a \right )\rightarrow \sum a^{2}\geq \sum a$




#434588 Giải hệ phương trình: $\left\{ \begin{array...

Đã gửi bởi trananh2771998 on 11-07-2013 - 20:28 trong Phương trình - hệ phương trình - bất phương trình

Từ phương trình thứ 2 của hệ nhân ra ta có

$x^{2}+4x+2y= 8$

$\rightarrow \left ( x^{2}+x \right )+\left ( 3x+2y \right )=8$

Và từ phương trình đầu của hệ tương đương với

$\left ( x^{2}+x \right )\left ( 3x+2y \right )=12$

Đặt

$x^{2}+x= a$

$3x+2y=b$

Ta có hệ mới

$\left\{\begin{matrix} ab=12 & & \\ a+b=8& & \end{matrix}\right.$.

Hệ này không khó nhỉ :icon6:




#433879 Hội những người độc thân thích chém gió !

Đã gửi bởi trananh2771998 on 09-07-2013 - 00:15 trong Góc giao lưu

Em xin một vé :ph34r: :ph34r:




#433023 $\sum \frac{a^{2}}{1+b^{3}...

Đã gửi bởi trananh2771998 on 05-07-2013 - 12:02 trong Bất đẳng thức và cực trị

Cho a,b,c dương thỏa mãn $\sum a^{4}=3$ Chứng minh

$\sum \frac{a^{2}}{1+b^{3}}\geq \frac{3}{2}$




#432889 tìm các số nguyên dương x,y,z thỏa mãn (x+1)(y+z)=xyz+2

Đã gửi bởi trananh2771998 on 04-07-2013 - 20:52 trong Các dạng toán khác

bạn làm cái gì vậy

Ý bạn ấy chắc  là đặt cho bài toán trở về dạng quen thuộc :icon6:




#432877 Cho $x+y=1$. Tìm GTLN của $\frac{x^3}{x+1...

Đã gửi bởi trananh2771998 on 04-07-2013 - 19:44 trong Bất đẳng thức và cực trị

mình thư chém xem được không nhé 

Ta thấy x+y=1 mà x,y  là số thực không âm nên $0\leq x,y\leq 1ta sẽ chứng minh

$\frac{x^{3}}{x+1}\leq \frac{x^{2}}{2}$(1)

Thật vậy ta có(1)$\Leftrightarrow 2x^{3}\leq x^{3}+x^{2}\Leftrightarrow x^{3}\leq x^{2}$ điều này hiển nhiên vì$0\leq x\leq 1$

Dấu bằng xảy ra khi x=1 hoặc x=0

tương tự$\frac{y^{3}}{y+1}\leq \frac{y^{2}}{2}$

$\Rightarrow P\leq \frac{x^{2}+y^{2}}{2}=\frac{1-2xy}{2}=\frac{1}{2}-xy\leq \frac{1}{2}$

Vậy MaxP=$\frac{1}{2}$ tại=1,y=0 hoặc x=0,y=1

Cách giải hay đó bạn. Làm thế nào để nghĩ ra được hướng đó vậy
 
 
 



#432422 $\left\{\begin{matrix} x^{2}-4y^...

Đã gửi bởi trananh2771998 on 02-07-2013 - 22:58 trong Phương trình, hệ phương trình và bất phương trình

Hệ pt đẳng cấp, đặt $y=ax$ là ra.

Giải chi tiết đi bạn.Đừng nói suông như thế




#430650 $\sum \frac{a+3}{(a+1)^2}\geqslant 3...

Đã gửi bởi trananh2771998 on 26-06-2013 - 00:38 trong Bất đẳng thức và cực trị

http://diendantoanho...fracz3z12geq-3/




#430458 $\frac{x+3}{(x+1)^{2}}+\frac...

Đã gửi bởi trananh2771998 on 25-06-2013 - 12:25 trong Bất đẳng thức và cực trị

Mình nghĩ là hướng có thể như thế này nhưng chưa chắc đúng

Ta có $\left ( x+1 \right )^{2}\leq 2\left ( x^{2}+1 \right )$

Từ đó bất đẳng thức cần chứng minh có dạng

$\sum \frac{x+3}{x^{2}+1}\geq 6$

:ukliam2:




#430123 Xấu nhưng biết phấn đấu

Đã gửi bởi trananh2771998 on 24-06-2013 - 00:52 trong Góc giao lưu

em thi trường ít điểm lắm hi vọng là đỗ :( em rất dốt môn toán

NLT là ai mà sao mọi người nhắc đến nhìu vậy

hihi không sao đâu em.CHỉ cần có cố gắng thôi mà .Cố lên em :namtay




#430114 Xấu nhưng biết phấn đấu

Đã gửi bởi trananh2771998 on 24-06-2013 - 00:29 trong Góc giao lưu

ảnh trên cp là gì ạ  :wacko:



Có nhưng em ms khóa được 1 tháng rùi thi xong đh em mới vào  :luoi:

hi hi em quyết tâm thật đấy.Chúc em vào đại học mà mình muốn nhé




#430111 Xấu nhưng biết phấn đấu

Đã gửi bởi trananh2771998 on 24-06-2013 - 00:21 trong Góc giao lưu

ừ không sao đâu em .Em có nick fb không cho anh xin cái :lol:




#429906 $f\left ( mf\left ( n \right ) \right )= nf\lef...

Đã gửi bởi trananh2771998 on 23-06-2013 - 00:08 trong Phương trình hàm

Do $f$ tăng thực sự nên $f$ đơn ánh.

Cho $P(m,n)$ có tính chất $f(mf(n))=nf(2m)$ với $m,n\in N^{*}$ và $f(2)=c$

$P(1,n)\Rightarrow f(f(n))=c \cdot n$

$P(1,1)\Rightarrow f(f(1))=f(2)\Rightarrow f(1)=2$

$P(1,2)\Rightarrow f(f(2))=2f(2)\Rightarrow f(c)=2c$

Ta có $f(f(n))=nf(2)\Rightarrow f(f(f(n)))=f(c \cdot n)\Rightarrow c\cdot f(n)=2f(2n)$

$P(f(m),n)\Rightarrow f(f(m)f(n))=nf(2f(m))=nmf(4)\Rightarrow f(f(f(m)f(n)))=f(mnf(4))$

$\Rightarrow c\cdot f(n)f(m)=4f(2mn)\Rightarrow f(m)f(n)=2f(mn)$

Đặt $f(n)=2g(n)$ ta có $g(mn)=g(m)g(n)$ và $g(c)=c$

Ta sẽ chứng minh $f(n)=2n$.

Theo qui nạp ta có $g(n^k)=(g(n))^k$ với $\forall k\in \mathbb{N^*}$ nên $g(c^k)=c^k$

Giả sử $g(n)\neq n$ thì luôn tồn tại cặp số $t,u\in \mathbb{N^*}$

Sao cho $2^t>n^u$ mà $c^t=g(c^t)<g(n^u)$ hoặc $c^t<n^u$ mà $c^t=g(c^t)>g(n^u)$

Mâu thuẫn với $g$ tăng (do $f$ tăng)

Nên $g(n)=n$ hay $f(n)=2n$

$KL:\boxed{f(n)=2n},\forall n\in \mathbb{N^*}$

Sao thím nghĩ ra được cái $f(2)=c$ vậy